1answer.
Ask question
Login Signup
Ask question
All categories
  • English
  • Mathematics
  • Social Studies
  • Business
  • History
  • Health
  • Geography
  • Biology
  • Physics
  • Chemistry
  • Computers and Technology
  • Arts
  • World Languages
  • Spanish
  • French
  • German
  • Advanced Placement (AP)
  • SAT
  • Medicine
  • Law
  • Engineering
Schach [20]
3 years ago
6

What is the area of the triangle in the diagram?

Mathematics
2 answers:
dsp733 years ago
5 0

Answer:

The answer is the 2nd one

Step-by-step explanation:

mariarad [96]3 years ago
5 0

Answer:

(B)          

Step-by-step explanation:

Given: A triangle whose coordinates are (0,0), (x_{1},y_{1}) and (x_{2},y_{1}).

To find: The area of the triangle.

Solution: We know that Area of triangle=\frac{1}{2}{\times}base{\times}height.

Now, Base=\sqrt{(x_{2}-x_{1})^{2}+(y_{1}-y_{1})^{2}}

Base=(x_{2}-x_{1})

And height=\sqrt{(0-0)^2+(y_{1}-0)^2}

Height= y_{1}

Therefore, area of triangle= \frac{1}{2}{\times}(x_{2}-x_{1})}{\times}y_{1}

Area of triangle=\frac{1}{2}y_{1}(x_{2}-x_{1})}

You might be interested in
How does the rate of change vary from point to point?
Tpy6a [65]

\bf \begin{array}{|cc|ll} \cline{1-2} x&y\\ \cline{1-2} 40&32\\ 28&16\\ 16&12\\ \cline{1-2} \end{array}~\hfill \stackrel{\textit{average rate of change}}{slope} \\\\[-0.35em] ~\dotfill\\\\ (\stackrel{x_1}{40}~,~\stackrel{y_1}{32})\qquad (\stackrel{x_2}{28}~,~\stackrel{y_2}{16}) \\\\\\ slope = m\implies \cfrac{\stackrel{rise}{ y_2- y_1}}{\stackrel{run}{ x_2- x_1}}\implies \cfrac{16-32}{28-40}\implies \cfrac{-16}{-12}\implies \boxed{\cfrac{4}{3}} \\\\[-0.35em] ~\dotfill

\bf (\stackrel{x_1}{28}~,~\stackrel{y_1}{16})\qquad (\stackrel{x_2}{16}~,~\stackrel{y_2}{12}) \\\\\\ slope = m\implies \cfrac{\stackrel{rise}{ y_2- y_1}}{\stackrel{run}{ x_2- x_1}}\implies \cfrac{12-16}{16-28}\implies \cfrac{-4}{-12}\implies \boxed{\cfrac{1}{3}}

8 0
3 years ago
Divide.
algol13

Answer:

Answer in simplified form is - 10 1/2 .

Step-by-step explanation:

We have given,

(5 1/4) ÷ (- 2 1/2)

This can be simplified as :

(5 1/4) ÷ ( -2 1/2)

Since 5 1/4 = 21/4  and -2 1/ 2 = -5/2

So we can write,

(5 1/4) ÷ ( -2 1/2) = 21/4 ÷ ( - 5/2)

or (21/4) / (-5/2)

or ( 21/4) * (-2/5)

or -42/20

or -21/10

or - 10 1/2 , this is the answer

Hence we get answer in simplified form as -10 1/2

7 0
3 years ago
Sixty-one diminished by ten x. Answer: _______________________ . The product of twice a number k. Answer: ______________________
KengaRu [80]

Answer:

61 - 10x

2k

42 / 6 = 7

38 + 2m

3y - 6

Step-by-step explanation:

A. Sixty-one diminished by ten x

= 61 - 10x

B. The product of twice a number k

= 2 * k

= 2k

C. The quotient of forty-two and six is seven

42 / 6 = 7

D. Twice the total of nineteen and a number m.

= 2(19 + m)

= 38 + 2m

E. Six less than 3 times y

3y - 6

6 0
3 years ago
Please help i’m in a hurry
Wittaler [7]

Answer:

the answer should be 18

Step-by-step explanation:

this isnt exactly hard when you get the first part with ST beeing 36 then you can just halve that and the answer is 18

6 0
3 years ago
A rectangle is inscribed with its base on the x-axis and its upper corners on the parabola y = 7 − x 2 y=7-x2. what are the dime
slega [8]
A rectangle is inscribed with its base on the x-axis and its upper corners on the parabola y = 7 − x 2 y=7-x2. what are the dimensions of such a rectangle with the greatest possible area?

5 0
3 years ago
Other questions:
  • ACDF is a trapezoid and
    15·1 answer
  • Morgan writes the expression
    6·1 answer
  • If Kim has 10 apples and jack has 10 strawberries how many fruits do they have
    7·1 answer
  • For your birthday you go to the movies with three friends. You buy 4 boxes of popcorn, one for you and one for each of your frie
    14·2 answers
  • Simplify: 3(r + 12r^2) – 2(r + 8)
    7·2 answers
  • PLEASE HELP FAST!!! ITS EASY BUT NOT FOR ME!!
    5·2 answers
  • Question 4 help me and i will give brainlist
    5·2 answers
  • The three are the whole question.
    15·1 answer
  • At a restaurant, you buy a combo meal for $7.50. You also want an ice cream sundae for dessert for $3.00. You brought $10 in cas
    15·2 answers
  • Is this a function ?
    12·1 answer
Add answer
Login
Not registered? Fast signup
Signup
Login Signup
Ask question!